Find Equation for Tangent Line to y=e^x at Origin

  • Thread starter Thread starter iRaid
  • Start date Start date
  • Tags Tags
    Tangent
Click For Summary
To find the equation of the tangent line to the graph of y=e^x that passes through the origin, the derivative y' = e^x is used to determine the slope at any point (x,y). The general form of the tangent line at a point x=x_0 is given by y = e^{x_0}(x - x_0) + e^{x_0}. To ensure the tangent line intersects the origin (0,0), the value of x_0 must be calculated, leading to the conclusion that x_0 cannot be zero. The discussion emphasizes the need to find the correct value of x_0 that allows the tangent line to pass through the origin.
iRaid
Messages
558
Reaction score
8

Homework Statement


Find an equation for a line that is tangent to the graph of y=ex and goes through the origin.

Homework Equations


The Attempt at a Solution


y'=ex

That's about all I can think of. I don't know how to make the tangent line go through the origin. Can someone lead me in the right direction?
 
Last edited:
Physics news on Phys.org
iRaid said:

Homework Statement


Find an equation for a line that is tangent to the grpah of y=ex and goes through the origin.


Homework Equations





The Attempt at a Solution


y'=ex

That's about all I can think of. I don't know how to make the tangent line go through the origin. Can someone lead me in the right direction?

At any point (x,y) on the graph, how would you compute the slope of the tangent line?

RGV
 
Ray Vickson said:
At any point (x,y) on the graph, how would you compute the slope of the tangent line?

RGV

Find the derivative at that point.
 
And you have already said that the derivative is again e^x.

So the tangent line to y= e^x at x= x_0 would be y= e^{x_0}(x- x_0)+ e^{x_0}. Now, what must x_0[/b] be so that goes through (0, 0)?
 
0? Not really sure..
 
Question: A clock's minute hand has length 4 and its hour hand has length 3. What is the distance between the tips at the moment when it is increasing most rapidly?(Putnam Exam Question) Answer: Making assumption that both the hands moves at constant angular velocities, the answer is ## \sqrt{7} .## But don't you think this assumption is somewhat doubtful and wrong?

Similar threads

  • · Replies 1 ·
Replies
1
Views
975
  • · Replies 6 ·
Replies
6
Views
2K
  • · Replies 11 ·
Replies
11
Views
5K
  • · Replies 7 ·
Replies
7
Views
2K
  • · Replies 6 ·
Replies
6
Views
2K
  • · Replies 4 ·
Replies
4
Views
1K
  • · Replies 15 ·
Replies
15
Views
5K
  • · Replies 1 ·
Replies
1
Views
1K
Replies
2
Views
1K
Replies
1
Views
4K